LSAT and Law School Admissions Forum

Get expert LSAT preparation and law school admissions advice from PowerScore Test Preparation.

User avatar
 Dave Killoran
PowerScore Staff
  • PowerScore Staff
  • Posts: 5852
  • Joined: Mar 25, 2011
|
#88211
Complete Question Explanation
(The complete setup for this game can be found here: lsat/viewtopic.php?f=172&p=88204#p88204)

The correct answer choice is (B)

Answer choice (A) can be eliminated because, as discussed during the game setup, when J is not fourth, J must be sixth.

Answer choice (B) is the correct answer.

Answer choice (C) can be eliminated because from the first rule either K or R must be second or third.

Answer choice (D) can be eliminated because from the third rule when J is fourth then S must have a higher cost than J.

Answer choice (E) can be eliminated because, as discussed during the game setup, when J is not fourth, J must be sixth.

Get the most out of your LSAT Prep Plus subscription.

Analyze and track your performance with our Testing and Analytics Package.